Difference between revisions of "1999 AIME Problems/Problem 13"

m
m (fmtting)
Line 1: Line 1:
 
== Problem ==
 
== Problem ==
Forty teams play a tournament in which every team plays every other team exactly once.  No ties occur, and each team has a <math>50 \%</math> chance of winning any game it plays.  The probability that no two teams win the same number of games is <math>m/n,</math> where <math>m_{}</math> and <math>n_{}</math> are relatively prime positive integers.  Find <math>\log_2 n.</math>
+
Forty teams play a tournament in which every team plays every other team exactly once.  No ties occur, and each team has a <math>50 \%</math> chance of winning any game it plays.  The [[probability]] that no two teams win the same number of games is <math>\frac mn,</math> where <math>m_{}</math> and <math>n_{}</math> are [[relatively prime]] positive integers.  Find <math>\log_2 n.</math>
  
 
== Solution ==
 
== Solution ==
 
Now one team must win 0 games, one team must win one game, ... , and one team must win all 39 games it plays. Now let's assume that team one wins all games, ... , and team 40 loses all games. The probability that team 40 wins all games is <math>\dfrac{1}{2^{39}}</math>. Now that team 39 has lost it's game, it has a probability of <math>\dfrac{1}{2^{38}}</math> of winning all of the other games. So on, until we get
 
Now one team must win 0 games, one team must win one game, ... , and one team must win all 39 games it plays. Now let's assume that team one wins all games, ... , and team 40 loses all games. The probability that team 40 wins all games is <math>\dfrac{1}{2^{39}}</math>. Now that team 39 has lost it's game, it has a probability of <math>\dfrac{1}{2^{38}}</math> of winning all of the other games. So on, until we get
  
<math>\frac{1}{2^{39+38+\cdots+1}}</math>
+
<cmath>\frac{1}{2^{39+38+\cdots+1}} = \frac{1}{2^{780}}</cmath>
  
 
But that was assuming that we knew which team got what score. Now we need to account of the permutations of each team:
 
But that was assuming that we knew which team got what score. Now we need to account of the permutations of each team:
  
<math>\dfrac{40!}{2^{780}}</math>
+
<cmath>\dfrac{40!}{2^{780}}</cmath>
  
Now we need to find how many 2's there are in 40! . There are
+
Now we need to find how many <math>2</math>'s there are in <math>40!</math>. There are
  
<math>\lfloor \dfrac{40}{2} \rfloor +\lfloor \dfrac{40}{4} \rfloor +\lfloor \dfrac{40}{8} \rfloor +\lfloor \dfrac{40}{16} \rfloor +\lfloor \dfrac{40}{32} \rfloor = 38</math>
+
<cmath>\left\lfloor \dfrac{40}{2} \right\rfloor +\left\lfloor\dfrac{40}{4} \right\rfloor +\left\lfloor\dfrac{40}{8} \right\rfloor +\left\lfloor\dfrac{40}{16} \right\rfloor +\left\lfloor\dfrac{40}{32} \right\rfloor = 38</cmath>
  
 
Therefore,
 
Therefore,
  
<math>n=2^{780-38}=2^{742}</math>
+
<cmath>n=2^{780-38}=2^{742}</cmath>
  
 
and
 
and
  
<math>\log_{2} n =\boxed{742}</math>
+
<cmath>\log_{2} n =\boxed{742}</cmath>
  
 
== See also ==
 
== See also ==

Revision as of 18:13, 28 October 2007

Problem

Forty teams play a tournament in which every team plays every other team exactly once. No ties occur, and each team has a $50 \%$ chance of winning any game it plays. The probability that no two teams win the same number of games is $\frac mn,$ where $m_{}$ and $n_{}$ are relatively prime positive integers. Find $\log_2 n.$

Solution

Now one team must win 0 games, one team must win one game, ... , and one team must win all 39 games it plays. Now let's assume that team one wins all games, ... , and team 40 loses all games. The probability that team 40 wins all games is $\dfrac{1}{2^{39}}$. Now that team 39 has lost it's game, it has a probability of $\dfrac{1}{2^{38}}$ of winning all of the other games. So on, until we get

\[\frac{1}{2^{39+38+\cdots+1}} = \frac{1}{2^{780}}\]

But that was assuming that we knew which team got what score. Now we need to account of the permutations of each team:

\[\dfrac{40!}{2^{780}}\]

Now we need to find how many $2$'s there are in $40!$. There are

\[\left\lfloor \dfrac{40}{2} \right\rfloor +\left\lfloor\dfrac{40}{4} \right\rfloor +\left\lfloor\dfrac{40}{8} \right\rfloor +\left\lfloor\dfrac{40}{16} \right\rfloor +\left\lfloor\dfrac{40}{32} \right\rfloor = 38\]

Therefore,

\[n=2^{780-38}=2^{742}\]

and

\[\log_{2} n =\boxed{742}\]

See also

1999 AIME (ProblemsAnswer KeyResources)
Preceded by
Problem 12
Followed by
Problem 14
1 2 3 4 5 6 7 8 9 10 11 12 13 14 15
All AIME Problems and Solutions